Proving a sequence is bounded from below











up vote
1
down vote

favorite












Let $$a_1=3quad , quad a_{n+1}=frac{3a_n}{4}+frac{1}{a_n}$$
Show that $a_n$ converges.



I know that I need to prove that $a_n$ is monotonic and bounded.



I've assumed that the limit exists, made the calculation and get that $L=2$, hence claiming two claims:





  1. $a_nge 2$ (I know that 0 is a simpler bound)


  2. $a_n$ is monotonically decreasing.


Now, if I can prove the first claim. then I can write $$a_{n+1}=frac{3a_n}{4}+frac{1}{a_n}=frac{3a_n}{4}+frac{a_n}{a_n^2}le frac{3a_n}{4}+frac{a_n}{4}=a_n$$ which proves the second claim.



My only problem is proving the first one. If it is not the right direction, please hint me.










share|cite|improve this question






















  • I think it oscillates above and below 2, but I might be wrong
    – mathworker21
    Nov 13 at 8:08










  • @mathworker21, it does not.
    – y12
    Nov 13 at 8:09










  • 1. You write "I've assumed that the limit exists, made the calculation and get that L=2". That is wrong. Show us your calculation.
    – miracle173
    Nov 13 at 9:32










  • 2. Why do you think you have to use monotonicity? There are convergent sequences that are no monotone.
    – miracle173
    Nov 13 at 9:33










  • @miracle173, let $displaystyle lim_{ntoinfty}a_n=L$, then applying limit to both sides gives the equation $$L=frac{3L}{4}+frac{1}{L}iff L^2=4iff L=pm 2$$ and clearly $a_n>0$, thus $L=2$. Also, I have not written the question as it given - it asks "show that the sequence is monotonic and bounded, thus converges".
    – y12
    Nov 13 at 10:25















up vote
1
down vote

favorite












Let $$a_1=3quad , quad a_{n+1}=frac{3a_n}{4}+frac{1}{a_n}$$
Show that $a_n$ converges.



I know that I need to prove that $a_n$ is monotonic and bounded.



I've assumed that the limit exists, made the calculation and get that $L=2$, hence claiming two claims:





  1. $a_nge 2$ (I know that 0 is a simpler bound)


  2. $a_n$ is monotonically decreasing.


Now, if I can prove the first claim. then I can write $$a_{n+1}=frac{3a_n}{4}+frac{1}{a_n}=frac{3a_n}{4}+frac{a_n}{a_n^2}le frac{3a_n}{4}+frac{a_n}{4}=a_n$$ which proves the second claim.



My only problem is proving the first one. If it is not the right direction, please hint me.










share|cite|improve this question






















  • I think it oscillates above and below 2, but I might be wrong
    – mathworker21
    Nov 13 at 8:08










  • @mathworker21, it does not.
    – y12
    Nov 13 at 8:09










  • 1. You write "I've assumed that the limit exists, made the calculation and get that L=2". That is wrong. Show us your calculation.
    – miracle173
    Nov 13 at 9:32










  • 2. Why do you think you have to use monotonicity? There are convergent sequences that are no monotone.
    – miracle173
    Nov 13 at 9:33










  • @miracle173, let $displaystyle lim_{ntoinfty}a_n=L$, then applying limit to both sides gives the equation $$L=frac{3L}{4}+frac{1}{L}iff L^2=4iff L=pm 2$$ and clearly $a_n>0$, thus $L=2$. Also, I have not written the question as it given - it asks "show that the sequence is monotonic and bounded, thus converges".
    – y12
    Nov 13 at 10:25













up vote
1
down vote

favorite









up vote
1
down vote

favorite











Let $$a_1=3quad , quad a_{n+1}=frac{3a_n}{4}+frac{1}{a_n}$$
Show that $a_n$ converges.



I know that I need to prove that $a_n$ is monotonic and bounded.



I've assumed that the limit exists, made the calculation and get that $L=2$, hence claiming two claims:





  1. $a_nge 2$ (I know that 0 is a simpler bound)


  2. $a_n$ is monotonically decreasing.


Now, if I can prove the first claim. then I can write $$a_{n+1}=frac{3a_n}{4}+frac{1}{a_n}=frac{3a_n}{4}+frac{a_n}{a_n^2}le frac{3a_n}{4}+frac{a_n}{4}=a_n$$ which proves the second claim.



My only problem is proving the first one. If it is not the right direction, please hint me.










share|cite|improve this question













Let $$a_1=3quad , quad a_{n+1}=frac{3a_n}{4}+frac{1}{a_n}$$
Show that $a_n$ converges.



I know that I need to prove that $a_n$ is monotonic and bounded.



I've assumed that the limit exists, made the calculation and get that $L=2$, hence claiming two claims:





  1. $a_nge 2$ (I know that 0 is a simpler bound)


  2. $a_n$ is monotonically decreasing.


Now, if I can prove the first claim. then I can write $$a_{n+1}=frac{3a_n}{4}+frac{1}{a_n}=frac{3a_n}{4}+frac{a_n}{a_n^2}le frac{3a_n}{4}+frac{a_n}{4}=a_n$$ which proves the second claim.



My only problem is proving the first one. If it is not the right direction, please hint me.







calculus






share|cite|improve this question













share|cite|improve this question











share|cite|improve this question




share|cite|improve this question










asked Nov 13 at 8:04









y12

324




324












  • I think it oscillates above and below 2, but I might be wrong
    – mathworker21
    Nov 13 at 8:08










  • @mathworker21, it does not.
    – y12
    Nov 13 at 8:09










  • 1. You write "I've assumed that the limit exists, made the calculation and get that L=2". That is wrong. Show us your calculation.
    – miracle173
    Nov 13 at 9:32










  • 2. Why do you think you have to use monotonicity? There are convergent sequences that are no monotone.
    – miracle173
    Nov 13 at 9:33










  • @miracle173, let $displaystyle lim_{ntoinfty}a_n=L$, then applying limit to both sides gives the equation $$L=frac{3L}{4}+frac{1}{L}iff L^2=4iff L=pm 2$$ and clearly $a_n>0$, thus $L=2$. Also, I have not written the question as it given - it asks "show that the sequence is monotonic and bounded, thus converges".
    – y12
    Nov 13 at 10:25


















  • I think it oscillates above and below 2, but I might be wrong
    – mathworker21
    Nov 13 at 8:08










  • @mathworker21, it does not.
    – y12
    Nov 13 at 8:09










  • 1. You write "I've assumed that the limit exists, made the calculation and get that L=2". That is wrong. Show us your calculation.
    – miracle173
    Nov 13 at 9:32










  • 2. Why do you think you have to use monotonicity? There are convergent sequences that are no monotone.
    – miracle173
    Nov 13 at 9:33










  • @miracle173, let $displaystyle lim_{ntoinfty}a_n=L$, then applying limit to both sides gives the equation $$L=frac{3L}{4}+frac{1}{L}iff L^2=4iff L=pm 2$$ and clearly $a_n>0$, thus $L=2$. Also, I have not written the question as it given - it asks "show that the sequence is monotonic and bounded, thus converges".
    – y12
    Nov 13 at 10:25
















I think it oscillates above and below 2, but I might be wrong
– mathworker21
Nov 13 at 8:08




I think it oscillates above and below 2, but I might be wrong
– mathworker21
Nov 13 at 8:08












@mathworker21, it does not.
– y12
Nov 13 at 8:09




@mathworker21, it does not.
– y12
Nov 13 at 8:09












1. You write "I've assumed that the limit exists, made the calculation and get that L=2". That is wrong. Show us your calculation.
– miracle173
Nov 13 at 9:32




1. You write "I've assumed that the limit exists, made the calculation and get that L=2". That is wrong. Show us your calculation.
– miracle173
Nov 13 at 9:32












2. Why do you think you have to use monotonicity? There are convergent sequences that are no monotone.
– miracle173
Nov 13 at 9:33




2. Why do you think you have to use monotonicity? There are convergent sequences that are no monotone.
– miracle173
Nov 13 at 9:33












@miracle173, let $displaystyle lim_{ntoinfty}a_n=L$, then applying limit to both sides gives the equation $$L=frac{3L}{4}+frac{1}{L}iff L^2=4iff L=pm 2$$ and clearly $a_n>0$, thus $L=2$. Also, I have not written the question as it given - it asks "show that the sequence is monotonic and bounded, thus converges".
– y12
Nov 13 at 10:25




@miracle173, let $displaystyle lim_{ntoinfty}a_n=L$, then applying limit to both sides gives the equation $$L=frac{3L}{4}+frac{1}{L}iff L^2=4iff L=pm 2$$ and clearly $a_n>0$, thus $L=2$. Also, I have not written the question as it given - it asks "show that the sequence is monotonic and bounded, thus converges".
– y12
Nov 13 at 10:25










3 Answers
3






active

oldest

votes

















up vote
3
down vote



accepted










To show boundedness from below, you may use AM-GM:
$$frac{3a_n}{4}+frac{1}{a_n}geq 2 sqrt{frac{3a_n}{4}cdot frac{1}{a_n}}= sqrt{3}$$
For convergence you may proceed as follows:





  • $f(x) = frac{3x}{4} + frac{1}{x}$ has a fixpoint for $x^{star} = 2$

  • $f'(x) = frac{3}{4}-frac{1}{x^2}$

  • A quick calculation shows that $|f'(x)| < 1$ for $x > frac{2}{sqrt{7}} Rightarrow |f'(x)| leq color{blue}{q} < 1$ for $x geq sqrt{3}$


So, for any starting value $a_0 > 0$ you get
$$|2-a_{n+1}| = |f'(xi_n)||2-a_n| < color{blue}{q} |2-a_n| < color{blue}{q^n}|2-a_1|stackrel{nto infty}{longrightarrow}0 $$



Edit after comment:



Specifically for your question concerning $a_n geq 2$:
$$ color{blue}{a_{n+1}-2} = frac{3a_n}{4}+frac{1}{a_n} - left(frac{3cdot 2}{4}+frac{1}{2} right)= frac{3}{4}left( a_n - 2 right) - frac{a_n - 2}{2a_n}$$
$$ = left(frac{3}{4} - frac{1}{2a_n} right)(a_n -2) stackrel{color{blue}{a_n geq 2}}{geq} frac{1}{2}(a_n -2) color{blue}{geq 0}$$






share|cite|improve this answer























  • "show that $a_n$ converges"
    – mathworker21
    Nov 13 at 8:08










  • A simpler lower bound for $a_n$ is $0$, but from that I cannot show that $a_n$ decreases...
    – y12
    Nov 13 at 8:10










  • @y12: Yes. But the lower bound $sqrt{3}$ is very useful for showing that the iteration converges for any starting value $a_0 > 0$ to the positive fixpoint of $f$, because $a_1 geq sqrt{3} > frac{2}{sqrt{7}}$.
    – trancelocation
    Nov 13 at 8:31










  • I'm not allowed to use functions...
    – y12
    Nov 13 at 8:44










  • @y12: I added something concerning your specific question.
    – trancelocation
    Nov 13 at 10:51


















up vote
3
down vote













The inequality $ x geq 2$ implies $frac {3x} 4+frac 1 x geq 2$ because$frac {3x} 4+frac 1 x$ is increasing on $x >frac 2 {sqrt3}$ (and $frac {3times 2} 4+frac 1 2 =2$). This gives (by induction) $a_n geq 2$ for all $n$.






share|cite|improve this answer























  • Kavi Rama.You show (nicely) that for $xge 2$ your function is increasing.But $a_n$ is decreasing . Could go below $2$? By AM-GM $ge √3$.Your thoughts , thanks, Peter m.wolframalpha.com/input/…
    – Peter Szilas
    Nov 13 at 11:19










  • No, it cannot. $a_n geq 2$ implies $frac {3a_n} 4+frac 1 {a_n} geq frac {(3)(2)} 4+frac 1 2 =2$. Use induction to conclude that every $a_n geq 2$.
    – Kavi Rama Murthy
    Nov 13 at 11:52












  • Kavi Rama.Thanks.
    – Peter Szilas
    Nov 13 at 12:32


















up vote
0
down vote













Let $a_n=2+d_n .$ Then $$d_{n+1}=3(2+d_n)/4+ 1/(2+d_n)-2.$$ Use some elementary algebra on this to show that $d_n>0implies d_n>d_{n+1}>0.$



So by induction on $n,$ if $0<a_1-2=d_1$ then $a_n$ decreases strictly to a limit $Lgeq 2.$






share|cite|improve this answer























  • You should get (assuming $d_nne -2$) that $d_{n+1}=d_n(frac {3}{4}-frac {1}{4+2d_n}).$ So if $d_n>0$ then $frac {3}{4}d_n>d_{n+1}>0.$
    – DanielWainfleet
    Nov 13 at 8:52













Your Answer





StackExchange.ifUsing("editor", function () {
return StackExchange.using("mathjaxEditing", function () {
StackExchange.MarkdownEditor.creationCallbacks.add(function (editor, postfix) {
StackExchange.mathjaxEditing.prepareWmdForMathJax(editor, postfix, [["$", "$"], ["\\(","\\)"]]);
});
});
}, "mathjax-editing");

StackExchange.ready(function() {
var channelOptions = {
tags: "".split(" "),
id: "69"
};
initTagRenderer("".split(" "), "".split(" "), channelOptions);

StackExchange.using("externalEditor", function() {
// Have to fire editor after snippets, if snippets enabled
if (StackExchange.settings.snippets.snippetsEnabled) {
StackExchange.using("snippets", function() {
createEditor();
});
}
else {
createEditor();
}
});

function createEditor() {
StackExchange.prepareEditor({
heartbeatType: 'answer',
convertImagesToLinks: true,
noModals: true,
showLowRepImageUploadWarning: true,
reputationToPostImages: 10,
bindNavPrevention: true,
postfix: "",
imageUploader: {
brandingHtml: "Powered by u003ca class="icon-imgur-white" href="https://imgur.com/"u003eu003c/au003e",
contentPolicyHtml: "User contributions licensed under u003ca href="https://creativecommons.org/licenses/by-sa/3.0/"u003ecc by-sa 3.0 with attribution requiredu003c/au003e u003ca href="https://stackoverflow.com/legal/content-policy"u003e(content policy)u003c/au003e",
allowUrls: true
},
noCode: true, onDemand: true,
discardSelector: ".discard-answer"
,immediatelyShowMarkdownHelp:true
});


}
});














 

draft saved


draft discarded


















StackExchange.ready(
function () {
StackExchange.openid.initPostLogin('.new-post-login', 'https%3a%2f%2fmath.stackexchange.com%2fquestions%2f2996466%2fproving-a-sequence-is-bounded-from-below%23new-answer', 'question_page');
}
);

Post as a guest















Required, but never shown

























3 Answers
3






active

oldest

votes








3 Answers
3






active

oldest

votes









active

oldest

votes






active

oldest

votes








up vote
3
down vote



accepted










To show boundedness from below, you may use AM-GM:
$$frac{3a_n}{4}+frac{1}{a_n}geq 2 sqrt{frac{3a_n}{4}cdot frac{1}{a_n}}= sqrt{3}$$
For convergence you may proceed as follows:





  • $f(x) = frac{3x}{4} + frac{1}{x}$ has a fixpoint for $x^{star} = 2$

  • $f'(x) = frac{3}{4}-frac{1}{x^2}$

  • A quick calculation shows that $|f'(x)| < 1$ for $x > frac{2}{sqrt{7}} Rightarrow |f'(x)| leq color{blue}{q} < 1$ for $x geq sqrt{3}$


So, for any starting value $a_0 > 0$ you get
$$|2-a_{n+1}| = |f'(xi_n)||2-a_n| < color{blue}{q} |2-a_n| < color{blue}{q^n}|2-a_1|stackrel{nto infty}{longrightarrow}0 $$



Edit after comment:



Specifically for your question concerning $a_n geq 2$:
$$ color{blue}{a_{n+1}-2} = frac{3a_n}{4}+frac{1}{a_n} - left(frac{3cdot 2}{4}+frac{1}{2} right)= frac{3}{4}left( a_n - 2 right) - frac{a_n - 2}{2a_n}$$
$$ = left(frac{3}{4} - frac{1}{2a_n} right)(a_n -2) stackrel{color{blue}{a_n geq 2}}{geq} frac{1}{2}(a_n -2) color{blue}{geq 0}$$






share|cite|improve this answer























  • "show that $a_n$ converges"
    – mathworker21
    Nov 13 at 8:08










  • A simpler lower bound for $a_n$ is $0$, but from that I cannot show that $a_n$ decreases...
    – y12
    Nov 13 at 8:10










  • @y12: Yes. But the lower bound $sqrt{3}$ is very useful for showing that the iteration converges for any starting value $a_0 > 0$ to the positive fixpoint of $f$, because $a_1 geq sqrt{3} > frac{2}{sqrt{7}}$.
    – trancelocation
    Nov 13 at 8:31










  • I'm not allowed to use functions...
    – y12
    Nov 13 at 8:44










  • @y12: I added something concerning your specific question.
    – trancelocation
    Nov 13 at 10:51















up vote
3
down vote



accepted










To show boundedness from below, you may use AM-GM:
$$frac{3a_n}{4}+frac{1}{a_n}geq 2 sqrt{frac{3a_n}{4}cdot frac{1}{a_n}}= sqrt{3}$$
For convergence you may proceed as follows:





  • $f(x) = frac{3x}{4} + frac{1}{x}$ has a fixpoint for $x^{star} = 2$

  • $f'(x) = frac{3}{4}-frac{1}{x^2}$

  • A quick calculation shows that $|f'(x)| < 1$ for $x > frac{2}{sqrt{7}} Rightarrow |f'(x)| leq color{blue}{q} < 1$ for $x geq sqrt{3}$


So, for any starting value $a_0 > 0$ you get
$$|2-a_{n+1}| = |f'(xi_n)||2-a_n| < color{blue}{q} |2-a_n| < color{blue}{q^n}|2-a_1|stackrel{nto infty}{longrightarrow}0 $$



Edit after comment:



Specifically for your question concerning $a_n geq 2$:
$$ color{blue}{a_{n+1}-2} = frac{3a_n}{4}+frac{1}{a_n} - left(frac{3cdot 2}{4}+frac{1}{2} right)= frac{3}{4}left( a_n - 2 right) - frac{a_n - 2}{2a_n}$$
$$ = left(frac{3}{4} - frac{1}{2a_n} right)(a_n -2) stackrel{color{blue}{a_n geq 2}}{geq} frac{1}{2}(a_n -2) color{blue}{geq 0}$$






share|cite|improve this answer























  • "show that $a_n$ converges"
    – mathworker21
    Nov 13 at 8:08










  • A simpler lower bound for $a_n$ is $0$, but from that I cannot show that $a_n$ decreases...
    – y12
    Nov 13 at 8:10










  • @y12: Yes. But the lower bound $sqrt{3}$ is very useful for showing that the iteration converges for any starting value $a_0 > 0$ to the positive fixpoint of $f$, because $a_1 geq sqrt{3} > frac{2}{sqrt{7}}$.
    – trancelocation
    Nov 13 at 8:31










  • I'm not allowed to use functions...
    – y12
    Nov 13 at 8:44










  • @y12: I added something concerning your specific question.
    – trancelocation
    Nov 13 at 10:51













up vote
3
down vote



accepted







up vote
3
down vote



accepted






To show boundedness from below, you may use AM-GM:
$$frac{3a_n}{4}+frac{1}{a_n}geq 2 sqrt{frac{3a_n}{4}cdot frac{1}{a_n}}= sqrt{3}$$
For convergence you may proceed as follows:





  • $f(x) = frac{3x}{4} + frac{1}{x}$ has a fixpoint for $x^{star} = 2$

  • $f'(x) = frac{3}{4}-frac{1}{x^2}$

  • A quick calculation shows that $|f'(x)| < 1$ for $x > frac{2}{sqrt{7}} Rightarrow |f'(x)| leq color{blue}{q} < 1$ for $x geq sqrt{3}$


So, for any starting value $a_0 > 0$ you get
$$|2-a_{n+1}| = |f'(xi_n)||2-a_n| < color{blue}{q} |2-a_n| < color{blue}{q^n}|2-a_1|stackrel{nto infty}{longrightarrow}0 $$



Edit after comment:



Specifically for your question concerning $a_n geq 2$:
$$ color{blue}{a_{n+1}-2} = frac{3a_n}{4}+frac{1}{a_n} - left(frac{3cdot 2}{4}+frac{1}{2} right)= frac{3}{4}left( a_n - 2 right) - frac{a_n - 2}{2a_n}$$
$$ = left(frac{3}{4} - frac{1}{2a_n} right)(a_n -2) stackrel{color{blue}{a_n geq 2}}{geq} frac{1}{2}(a_n -2) color{blue}{geq 0}$$






share|cite|improve this answer














To show boundedness from below, you may use AM-GM:
$$frac{3a_n}{4}+frac{1}{a_n}geq 2 sqrt{frac{3a_n}{4}cdot frac{1}{a_n}}= sqrt{3}$$
For convergence you may proceed as follows:





  • $f(x) = frac{3x}{4} + frac{1}{x}$ has a fixpoint for $x^{star} = 2$

  • $f'(x) = frac{3}{4}-frac{1}{x^2}$

  • A quick calculation shows that $|f'(x)| < 1$ for $x > frac{2}{sqrt{7}} Rightarrow |f'(x)| leq color{blue}{q} < 1$ for $x geq sqrt{3}$


So, for any starting value $a_0 > 0$ you get
$$|2-a_{n+1}| = |f'(xi_n)||2-a_n| < color{blue}{q} |2-a_n| < color{blue}{q^n}|2-a_1|stackrel{nto infty}{longrightarrow}0 $$



Edit after comment:



Specifically for your question concerning $a_n geq 2$:
$$ color{blue}{a_{n+1}-2} = frac{3a_n}{4}+frac{1}{a_n} - left(frac{3cdot 2}{4}+frac{1}{2} right)= frac{3}{4}left( a_n - 2 right) - frac{a_n - 2}{2a_n}$$
$$ = left(frac{3}{4} - frac{1}{2a_n} right)(a_n -2) stackrel{color{blue}{a_n geq 2}}{geq} frac{1}{2}(a_n -2) color{blue}{geq 0}$$







share|cite|improve this answer














share|cite|improve this answer



share|cite|improve this answer








edited Nov 13 at 10:50

























answered Nov 13 at 8:07









trancelocation

8,1291519




8,1291519












  • "show that $a_n$ converges"
    – mathworker21
    Nov 13 at 8:08










  • A simpler lower bound for $a_n$ is $0$, but from that I cannot show that $a_n$ decreases...
    – y12
    Nov 13 at 8:10










  • @y12: Yes. But the lower bound $sqrt{3}$ is very useful for showing that the iteration converges for any starting value $a_0 > 0$ to the positive fixpoint of $f$, because $a_1 geq sqrt{3} > frac{2}{sqrt{7}}$.
    – trancelocation
    Nov 13 at 8:31










  • I'm not allowed to use functions...
    – y12
    Nov 13 at 8:44










  • @y12: I added something concerning your specific question.
    – trancelocation
    Nov 13 at 10:51


















  • "show that $a_n$ converges"
    – mathworker21
    Nov 13 at 8:08










  • A simpler lower bound for $a_n$ is $0$, but from that I cannot show that $a_n$ decreases...
    – y12
    Nov 13 at 8:10










  • @y12: Yes. But the lower bound $sqrt{3}$ is very useful for showing that the iteration converges for any starting value $a_0 > 0$ to the positive fixpoint of $f$, because $a_1 geq sqrt{3} > frac{2}{sqrt{7}}$.
    – trancelocation
    Nov 13 at 8:31










  • I'm not allowed to use functions...
    – y12
    Nov 13 at 8:44










  • @y12: I added something concerning your specific question.
    – trancelocation
    Nov 13 at 10:51
















"show that $a_n$ converges"
– mathworker21
Nov 13 at 8:08




"show that $a_n$ converges"
– mathworker21
Nov 13 at 8:08












A simpler lower bound for $a_n$ is $0$, but from that I cannot show that $a_n$ decreases...
– y12
Nov 13 at 8:10




A simpler lower bound for $a_n$ is $0$, but from that I cannot show that $a_n$ decreases...
– y12
Nov 13 at 8:10












@y12: Yes. But the lower bound $sqrt{3}$ is very useful for showing that the iteration converges for any starting value $a_0 > 0$ to the positive fixpoint of $f$, because $a_1 geq sqrt{3} > frac{2}{sqrt{7}}$.
– trancelocation
Nov 13 at 8:31




@y12: Yes. But the lower bound $sqrt{3}$ is very useful for showing that the iteration converges for any starting value $a_0 > 0$ to the positive fixpoint of $f$, because $a_1 geq sqrt{3} > frac{2}{sqrt{7}}$.
– trancelocation
Nov 13 at 8:31












I'm not allowed to use functions...
– y12
Nov 13 at 8:44




I'm not allowed to use functions...
– y12
Nov 13 at 8:44












@y12: I added something concerning your specific question.
– trancelocation
Nov 13 at 10:51




@y12: I added something concerning your specific question.
– trancelocation
Nov 13 at 10:51










up vote
3
down vote













The inequality $ x geq 2$ implies $frac {3x} 4+frac 1 x geq 2$ because$frac {3x} 4+frac 1 x$ is increasing on $x >frac 2 {sqrt3}$ (and $frac {3times 2} 4+frac 1 2 =2$). This gives (by induction) $a_n geq 2$ for all $n$.






share|cite|improve this answer























  • Kavi Rama.You show (nicely) that for $xge 2$ your function is increasing.But $a_n$ is decreasing . Could go below $2$? By AM-GM $ge √3$.Your thoughts , thanks, Peter m.wolframalpha.com/input/…
    – Peter Szilas
    Nov 13 at 11:19










  • No, it cannot. $a_n geq 2$ implies $frac {3a_n} 4+frac 1 {a_n} geq frac {(3)(2)} 4+frac 1 2 =2$. Use induction to conclude that every $a_n geq 2$.
    – Kavi Rama Murthy
    Nov 13 at 11:52












  • Kavi Rama.Thanks.
    – Peter Szilas
    Nov 13 at 12:32















up vote
3
down vote













The inequality $ x geq 2$ implies $frac {3x} 4+frac 1 x geq 2$ because$frac {3x} 4+frac 1 x$ is increasing on $x >frac 2 {sqrt3}$ (and $frac {3times 2} 4+frac 1 2 =2$). This gives (by induction) $a_n geq 2$ for all $n$.






share|cite|improve this answer























  • Kavi Rama.You show (nicely) that for $xge 2$ your function is increasing.But $a_n$ is decreasing . Could go below $2$? By AM-GM $ge √3$.Your thoughts , thanks, Peter m.wolframalpha.com/input/…
    – Peter Szilas
    Nov 13 at 11:19










  • No, it cannot. $a_n geq 2$ implies $frac {3a_n} 4+frac 1 {a_n} geq frac {(3)(2)} 4+frac 1 2 =2$. Use induction to conclude that every $a_n geq 2$.
    – Kavi Rama Murthy
    Nov 13 at 11:52












  • Kavi Rama.Thanks.
    – Peter Szilas
    Nov 13 at 12:32













up vote
3
down vote










up vote
3
down vote









The inequality $ x geq 2$ implies $frac {3x} 4+frac 1 x geq 2$ because$frac {3x} 4+frac 1 x$ is increasing on $x >frac 2 {sqrt3}$ (and $frac {3times 2} 4+frac 1 2 =2$). This gives (by induction) $a_n geq 2$ for all $n$.






share|cite|improve this answer














The inequality $ x geq 2$ implies $frac {3x} 4+frac 1 x geq 2$ because$frac {3x} 4+frac 1 x$ is increasing on $x >frac 2 {sqrt3}$ (and $frac {3times 2} 4+frac 1 2 =2$). This gives (by induction) $a_n geq 2$ for all $n$.







share|cite|improve this answer














share|cite|improve this answer



share|cite|improve this answer








edited Nov 13 at 16:50









miracle173

7,28822247




7,28822247










answered Nov 13 at 8:20









Kavi Rama Murthy

40.7k31751




40.7k31751












  • Kavi Rama.You show (nicely) that for $xge 2$ your function is increasing.But $a_n$ is decreasing . Could go below $2$? By AM-GM $ge √3$.Your thoughts , thanks, Peter m.wolframalpha.com/input/…
    – Peter Szilas
    Nov 13 at 11:19










  • No, it cannot. $a_n geq 2$ implies $frac {3a_n} 4+frac 1 {a_n} geq frac {(3)(2)} 4+frac 1 2 =2$. Use induction to conclude that every $a_n geq 2$.
    – Kavi Rama Murthy
    Nov 13 at 11:52












  • Kavi Rama.Thanks.
    – Peter Szilas
    Nov 13 at 12:32


















  • Kavi Rama.You show (nicely) that for $xge 2$ your function is increasing.But $a_n$ is decreasing . Could go below $2$? By AM-GM $ge √3$.Your thoughts , thanks, Peter m.wolframalpha.com/input/…
    – Peter Szilas
    Nov 13 at 11:19










  • No, it cannot. $a_n geq 2$ implies $frac {3a_n} 4+frac 1 {a_n} geq frac {(3)(2)} 4+frac 1 2 =2$. Use induction to conclude that every $a_n geq 2$.
    – Kavi Rama Murthy
    Nov 13 at 11:52












  • Kavi Rama.Thanks.
    – Peter Szilas
    Nov 13 at 12:32
















Kavi Rama.You show (nicely) that for $xge 2$ your function is increasing.But $a_n$ is decreasing . Could go below $2$? By AM-GM $ge √3$.Your thoughts , thanks, Peter m.wolframalpha.com/input/…
– Peter Szilas
Nov 13 at 11:19




Kavi Rama.You show (nicely) that for $xge 2$ your function is increasing.But $a_n$ is decreasing . Could go below $2$? By AM-GM $ge √3$.Your thoughts , thanks, Peter m.wolframalpha.com/input/…
– Peter Szilas
Nov 13 at 11:19












No, it cannot. $a_n geq 2$ implies $frac {3a_n} 4+frac 1 {a_n} geq frac {(3)(2)} 4+frac 1 2 =2$. Use induction to conclude that every $a_n geq 2$.
– Kavi Rama Murthy
Nov 13 at 11:52






No, it cannot. $a_n geq 2$ implies $frac {3a_n} 4+frac 1 {a_n} geq frac {(3)(2)} 4+frac 1 2 =2$. Use induction to conclude that every $a_n geq 2$.
– Kavi Rama Murthy
Nov 13 at 11:52














Kavi Rama.Thanks.
– Peter Szilas
Nov 13 at 12:32




Kavi Rama.Thanks.
– Peter Szilas
Nov 13 at 12:32










up vote
0
down vote













Let $a_n=2+d_n .$ Then $$d_{n+1}=3(2+d_n)/4+ 1/(2+d_n)-2.$$ Use some elementary algebra on this to show that $d_n>0implies d_n>d_{n+1}>0.$



So by induction on $n,$ if $0<a_1-2=d_1$ then $a_n$ decreases strictly to a limit $Lgeq 2.$






share|cite|improve this answer























  • You should get (assuming $d_nne -2$) that $d_{n+1}=d_n(frac {3}{4}-frac {1}{4+2d_n}).$ So if $d_n>0$ then $frac {3}{4}d_n>d_{n+1}>0.$
    – DanielWainfleet
    Nov 13 at 8:52

















up vote
0
down vote













Let $a_n=2+d_n .$ Then $$d_{n+1}=3(2+d_n)/4+ 1/(2+d_n)-2.$$ Use some elementary algebra on this to show that $d_n>0implies d_n>d_{n+1}>0.$



So by induction on $n,$ if $0<a_1-2=d_1$ then $a_n$ decreases strictly to a limit $Lgeq 2.$






share|cite|improve this answer























  • You should get (assuming $d_nne -2$) that $d_{n+1}=d_n(frac {3}{4}-frac {1}{4+2d_n}).$ So if $d_n>0$ then $frac {3}{4}d_n>d_{n+1}>0.$
    – DanielWainfleet
    Nov 13 at 8:52















up vote
0
down vote










up vote
0
down vote









Let $a_n=2+d_n .$ Then $$d_{n+1}=3(2+d_n)/4+ 1/(2+d_n)-2.$$ Use some elementary algebra on this to show that $d_n>0implies d_n>d_{n+1}>0.$



So by induction on $n,$ if $0<a_1-2=d_1$ then $a_n$ decreases strictly to a limit $Lgeq 2.$






share|cite|improve this answer














Let $a_n=2+d_n .$ Then $$d_{n+1}=3(2+d_n)/4+ 1/(2+d_n)-2.$$ Use some elementary algebra on this to show that $d_n>0implies d_n>d_{n+1}>0.$



So by induction on $n,$ if $0<a_1-2=d_1$ then $a_n$ decreases strictly to a limit $Lgeq 2.$







share|cite|improve this answer














share|cite|improve this answer



share|cite|improve this answer








edited Nov 13 at 8:55

























answered Nov 13 at 8:41









DanielWainfleet

33.4k31647




33.4k31647












  • You should get (assuming $d_nne -2$) that $d_{n+1}=d_n(frac {3}{4}-frac {1}{4+2d_n}).$ So if $d_n>0$ then $frac {3}{4}d_n>d_{n+1}>0.$
    – DanielWainfleet
    Nov 13 at 8:52




















  • You should get (assuming $d_nne -2$) that $d_{n+1}=d_n(frac {3}{4}-frac {1}{4+2d_n}).$ So if $d_n>0$ then $frac {3}{4}d_n>d_{n+1}>0.$
    – DanielWainfleet
    Nov 13 at 8:52


















You should get (assuming $d_nne -2$) that $d_{n+1}=d_n(frac {3}{4}-frac {1}{4+2d_n}).$ So if $d_n>0$ then $frac {3}{4}d_n>d_{n+1}>0.$
– DanielWainfleet
Nov 13 at 8:52






You should get (assuming $d_nne -2$) that $d_{n+1}=d_n(frac {3}{4}-frac {1}{4+2d_n}).$ So if $d_n>0$ then $frac {3}{4}d_n>d_{n+1}>0.$
– DanielWainfleet
Nov 13 at 8:52




















 

draft saved


draft discarded



















































 


draft saved


draft discarded














StackExchange.ready(
function () {
StackExchange.openid.initPostLogin('.new-post-login', 'https%3a%2f%2fmath.stackexchange.com%2fquestions%2f2996466%2fproving-a-sequence-is-bounded-from-below%23new-answer', 'question_page');
}
);

Post as a guest















Required, but never shown





















































Required, but never shown














Required, but never shown












Required, but never shown







Required, but never shown

































Required, but never shown














Required, but never shown












Required, but never shown







Required, but never shown







Popular posts from this blog

Biblatex bibliography style without URLs when DOI exists (in Overleaf with Zotero bibliography)

ComboBox Display Member on multiple fields

Is it possible to collect Nectar points via Trainline?